GameBoy 1-600 Full Version PDF
Document Details
Tags
Summary
This document appears to be a collection of medical case studies, featuring multiple-choice questions (MCQs) and answers. The cases cover various medical conditions, from gout to systemic lupus erythematosus. The questions and answers evaluate knowledge of clinical diagnoses and management.
Full Transcript
0001. A 40-year-old man with a history of recurrent painful attacks affecting the right big toe, was diagnosed with gout and started on allopurinol. He is also a known case of psoriasis for several years. He presented to the clinic with recurrent joint pain affecting the right big toe, left ankle,...
0001. A 40-year-old man with a history of recurrent painful attacks affecting the right big toe, was diagnosed with gout and started on allopurinol. He is also a known case of psoriasis for several years. He presented to the clinic with recurrent joint pain affecting the right big toe, left ankle, and bilateral knees. He is using allopurinol regularly as prescribed but has no improvement in his joint pain. Examinations revealed he has active arthritis at the left ankle and knee joints bilaterally (see lab results). Which of the following is the most likely cause for his joint pain? A. Active pseudogout arthritis. B. Active gout arthritis. C. Psoriatic arthritis. D. Osteoarthritis. Answer: C 0002. A 28-year-old woman with systemic lupus erythematosus for 2 years on hydroxychloroquine 200 mg/day, prednisone 5 mg/day, and azathioprine 100 mg/day presented to the Emergency Room with sudden onset of lower limb weakness and urinary incontinence. Examination revealed she has paraplegia and hyperreflexia (see lab results). Which of the following is the most appropriate test to be done immediately? A. Lumbar puncture, MRI brain B. Lumbar puncture, CT brain C. Lumbar puncture, MRI spinal cord D. Lumbar puncture, MRI, MRA, and MRV of the brain Answer: C 0003. A 50-year-old man is complaining of pain affecting the wrist joints bilaterally, second and third proximal interphalangeal joints, and right knee joint for the last 4 months. His examination revealed active arthritis at both wrists and second and third proximal interphalangeal joints (see lab results). Which of the following is the most appropriate treatment? A. Methotrexate B. Methotrexate and sulfasalazine C. Methotrexate and hydroxychloroquine D. Methotrexate, hydroxychloroquine, and prednisone Answer: D 0004. A 30-year-old woman with systemic lupus erythematosus is evaluated for right hip pain that started 3 weeks ago and occurs at night and with walking. Her current medications are azathioprine, hydroxychloroquine, prednisolone, and lisinopril for the last 5 years. Her examination was normal except for pain on internal rotation of the right hip. Laboratory tests including pelvic and left hip radiographs were normal. Which of the following is the most likely cause of hip pain? A. Active lupus arthritis B. Septic arthritis affecting right hip C. Avascular necrosis affecting right hip D. Degenerative changes affecting right hip Answer: C 0005. A 60-year-old man complaining of right knee pain for several years that has increased over the last few months. He also has noticed frequent knee clicks. A diagnosis of osteoarthritis is suspected. Which of the following is the most appropriate to confirm the diagnosis? A. Bilateral knee X-rays. B. MRI of the right knee. C. No further investigation. D. Arthroscopy for the right knee. Answer: C 0006. A 52-year-old man with rheumatoid arthritis for 12 years. His current treatment consists of methotrexate 15 mg weekly and adalimumab 40 mg/2 weeks. He presented to the clinic complaining of fever and shortness of breath for 1 week. His chest examination revealed coarse crackles over the lower part of the right lung (see lab results). Which of the following is the most appropriate action to be taken on this visit? A. Hold adalimumab and start antibiotics B. Hold methotrexate and start antibiotics C. Hold his current treatment and start antibiotics D. Continue his current treatment and start antibiotics Answer: C 0007. A 30-year-old man presented to the Emergency Department with an acute monoarthritis where joint aspiration was performed. Based on the presence of crystals in the synovial fluid, he was diagnosed with gout. Which of the following is the most likely type of the crystals seen? A. Needle-shaped, negatively birefringent B. Needle-shaped, weakly positively birefringent C. Rhomboidal-shaped, negatively birefringent D. Rhomboidal-shaped, weakly positively birefringent Answer: A 0008. A 30-year-old woman with a diagnosis of systemic lupus erythematosus who was maintained on mycophenolate mofetil 1 gm/day, hydroxychloroquine 200 mg/day, and prednisone 5 mg/day for the last 1 year. She has had inactive disease for more than 6 months. She is planning to get pregnant (see lab results). Which of the following is the most appropriate action at this time? A. Discontinue prednisone and start cyclosporine B. Discontinue hydroxychloroquine and start quinacrine C. Discontinue mycophenolate mofetil and start azathioprine D. Discontinue mycophenolate mofetil and start methotrexate Answer: C 0009. A 61-year-old woman is evaluated for a left pulmonary nodule that was discovered incidentally 2 months ago. She is currently asymptomatic and has not had shortness of breath, fever, chills, weight loss, or night sweats. Medical history is otherwise unremarkable, and she takes no medications. She never smoked. Examination revealed no cervical or supraclavicular lymphadenopathy, and the lungs are clear to auscultation (see report). CT scan chest: Showed an 8 mm left lung nodule. Which of the following is the most appropriate next step in management? A. PET/CT scan B. Bronchoalveolar lavage C. Transthoracic lung biopsy D. Observation and regular follow-up Answer: D 0010. A 63-year-old woman presented with exertional dyspnea for 8 months. She was diagnosed with COPD 3 years ago and has had 4 exacerbations in the last year. She completed pulmonary rehabilitation and quit smoking 2 years ago. Medications include long-acting B2 agonist, inhaled glucocorticoid, roflumilast, and albuterol as needed. Examination revealed no jugular venous distention; decreased breath sounds, a loud pulmonary component of S1 (see lab results and reports). Chest radiograph: Normal. Echocardiogram: Ejection fraction of 60%, no valvular or wall motion abnormalities, and estimated mean pulmonary artery pressure is 55 mmHg. Which of the following is the most appropriate treatment? A. Daily prednisone B. Long-term oxygen therapy C. Overnight pulse oximetry D. Repeat pulmonary rehabilitation Answer: B 0011. A 58-year-old woman presented to the medical clinic complaining of daily wheezing and breathlessness during allergy season. These episodes of wheezing have limited her activities several times a week. She has a long history of seasonal allergies in the form of itchy eyes and runny nose. No other medical history. Medications include loratadine as needed, and antihistamine eye drops. Examination revealed watery eyes with conjunctival irritation, expiratory wheezing, and normal cardiac examination (see reports). Chest radiograph: Unremarkable. Spirometry: FEV1 of 74% of predicted, which improved by 18% with a bronchodilator. Which of the following is the most appropriate treatment? A. Add a leukotriene antagonist B. Add a low-dose inhaled glucocorticoid C. Recommend daily oral antihistamine use D. Add a low-dose inhaled glucocorticoid and long-acting B2 agonist Answer: D 0012. A 71-year-old man is evaluated after a recent hospitalization 10 days ago for pneumonia and was discharged with a 10-day course of appropriate antibiotics. For the past 5 days, he has had a low-grade fever and increased shortness of breath. Pulmonary examination reveals decreased fremitus, dullness to percussion, and decreased breath sounds over the lower third of the right hemithorax (see lab results and report). Chest radiograph: Moderate right-sided pleural effusion. Which of the following is the most appropriate management? A. Ceftriaxone and azithromycin B. Chest tube and levofloxacin C. Repeat chest radiograph in 2 weeks D. Small-bore pleural drain and piperacillin-tazobactam Answer: D 0013. A 20-year-old man is evaluated for a 6-month history of chest and throat tightness, acute episodes of stridor, and a prolonged wheeze that occur during his college basketball practice. He has a history of moderate persistent asthma, which is controlled. He takes a short-acting B2 agonist inhaler as needed, but this does not relieve his symptoms. In addition to a low-dose inhaled glucocorticoid and a long-acting inhaled B2 agonist, he otherwise feels well. Medical history is otherwise unremarkable. Examination reveals normal vital signs and unremarkable systemic examination (see reports). Chest radiograph: Normal. Spirometry: No evidence of obstruction. Which of the following is the most appropriate next step in management? A. Echocardiography B. Allergen immunotherapy C. Otolaryngology evaluation D. Switch to a medium-dose inhaled glucocorticoid Answer: C 0014. A 33-year-old man presented with shortness of breath, fatigue, and nighttime cough. He has been working in a car repair shop for the past 1 year and notes that his symptoms began about 8 months ago. He believes his symptoms are associated with his workdays. He is being evaluated after several days off from work and currently has no symptoms or medical concerns. He has an unremarkable medical history and takes no medications. He has never smoked. Physical examination is unremarkable. A chest radiograph and spirometry results are normal. Which of the following is the most appropriate next step in management? A. High-resolution chest CT B. Inhaled glucocorticoid daily C. Advise him to switch employment D. Repeat spirometry after workplace exposure Answer: D 0015. A 49-year-old woman is evaluated for a 4-month history of dry cough, which began 4 months ago when she noted a paroxysmal onset of persistent coughing spells that make it difficult to carry on a conversation. She was initially diagnosed with allergic rhinitis and was started on a glucocorticoid nasal inhaler with no improvement. Medical history is otherwise unremarkable except for a history of dust allergies. Examination revealed normal vital signs, slight nasal congestion, and dry cough are noted during the examination. Cardiovascular, pulmonary, and abdominal examination findings are unremarkable (see reports). Chest radiograph: Normal. Spirometry: Slightly reduced FEV/FVC ratio of 70% which increased after bronchodilator challenge by 20%. Which of the following is the most appropriate diagnostic test to perform next? A. Allergy skin testing B. High-resolution chest CT C. Measurement of serum IgE levels D. Methacholine challenge testing Answer: D 0016. A 57-year-old man is evaluated for chronic cough, occasional wheezing, and exertional dyspnea while walking. Medical history includes an episode of bronchitis, for which he underwent outpatient treatment 8 months ago. He has a 20-pack-year smoking history. Examination revealed normal vital signs, mildly decreased breath sounds throughout both lungs, and scattered expiratory wheezes bilaterally. Spirometry shows an FEV1 of 75% of predicted and a post-bronchodilator FEV/FVC ratio of 68%. His modified Medical Research Council (mMRC) symptom score is 2. In addition to smoking cessation, which of the following is the most appropriate long-term management plan? A. Combination inhaled glucocorticoid and a long-acting bronchodilator B. Phosphodiesterase-4 inhibitor and combination inhaled glucocorticoid and long-acting bronchodilator C. Short-acting bronchodilator as needed, a long-acting bronchodilator, and pulmonary rehabilitation D. Short-acting bronchodilator as needed and an inhaled glucocorticoid Answer: C 0017. A 42-year-old woman presented to the clinic for follow-up of a 7-month history of chronic, nonproductive cough. She was recently diagnosed with cough-variant asthma after a methacholine challenge test. She was started on inhaled fluticasone and as-needed albuterol 2 months ago. Her cough initially improved; however, her symptoms have not fully resolved and are worsened at times, most notably when she lies down. She also noted that mild hoarseness occasionally accompanies her cough, especially when she wakes up in the morning. Examination revealed a dry cough with frequent throat clearing. The chest was clear with no wheezing. Which of the following is the most appropriate management? A. Add a long-acting B2-agonist B. Add a proton pump inhibitor C. Start nocturnal antitussive therapy D. Repeat methacholine challenge testing Answer: B 0018. A 17-year-old man presented to the Emergency Department with severe asthma exacerbation that began 12 hours ago. He is on an albuterol inhaler and a long-acting glucocorticoid at home without improvement. He was hospitalized and intubated 2 years ago. Examination revealed respiratory distress with diffuse expiratory wheezes. After 1 hour of systemic glucocorticoid and B2 agonist nebulizer, he still appeared in respiratory distress (see lab results and report). Chest X-ray: Hyperinflated lungs. Which of the following is the most appropriate next step in management? A. Admit to the ICU B. Reassure him and discharge home C. Admit to the general medical floor D. Discharge home the next day with follow-up Answer: A 0019. A 63-year-old man was evaluated in the Emergency Room for 1 week of progressive shortness of breath and found to have right pleural effusion on chest X-ray. Aspiration yielded a clear transudative fluid. Which of the following is the most likely cause? A. Pancreatitis B. Pneumonia C. Heart failure D. Malignancy Answer: C 0020. A 24-year-old woman is reviewed in the Asthma Clinic. She currently uses a salbutamol inhaler 100 mcg prn combined with a beclometasone dipropionate inhaler 400 mcg bd. Despite this, she is having frequent exacerbations of her asthma that required frequent courses of prednisolone and hospitalization three times in the last 6 months. What is the most appropriate next step in management? A. Switch beclometasone to fluticasone B. Start to take the salbutamol regularly C. Add long-acting B2 agonist (Salmeterol) D. Add long-acting anticholinergic bronchodilator (Tiotropium) Answer: C 0021. A 63-year-old woman is in the COPD clinic for frequent exacerbations, despite being maintained on a salbutamol inhaler as required and a tiotropium inhaler regularly. She was diagnosed with COPD three years ago and recently quit smoking. Her latest FEV1 was 39% of predicted. Which of the following is the most appropriate next step in her management? A. Salmeterol inhaler B. Betamethasone inhaler C. Long-term oxygen therapy D. Combined salmeterol + fluticasone inhaler Answer: D 0022. A 69-year-old smoker presents with a 3-week history of worsening breathlessness. Examination revealed normal vital signs and decreased breathing sounds over the lower left zone (see reports). Chest X-ray: Large left-sided pleural effusion. Pleural tapping: Pending result. Which of the following will most likely indicate that it is an exudative effusion? A. Pleural fluid LDH / Serum LDH < 0.6 B. Pleural fluid protein / Serum protein < 0.5 C. Effusion LDH level greater than 1/3rd the upper limit of serum LDH D. Effusion LDH level greater than 2/3rds the upper limit of serum LDH Answer: D 0023. A 33-year-old man with a history of severe asthma is admitted to the Emergency Department with an asthma attack. He received high-flow oxygen and nebulized salbutamol. He is unable to complete sentences, has bilateral expiratory wheeze, and is unable to perform a peak flow reading. He was immediately given intravenous hydrocortisone and nebulized salbutamol continuously. After 10 minutes, there is no improvement, and intravenous magnesium sulfate was given. His condition continues to deteriorate (see lab results). Which of the following is the most appropriate management? A. Non-invasive ventilation B. Add intravenous aminophylline C. Add nebulized ipratropium bromide D. Intubation and mechanical ventilation Answer: D 0024. A 34-year-old woman with a history of asthma and allergic rhinitis has been discharged from the hospital 2 weeks ago following an acute exacerbation of bronchial asthma and reports generally poor control with a persistent nighttime cough and exertional wheeze. Her current asthma therapy is salbutamol inhaler 100 mcg prn, beclometasone dipropionate inhaler 800 mcg bd, and salmeterol 50 mcg bd. Which of the following is the most appropriate next step in outpatient management? A. Ipratropium inhaler B. Low-dose oral prednisolone C. Leukotriene receptor antagonist D. Modified-release theophylline Answer: C 0025. A 33-year-old woman has been assessed in the Emergency Department with an acute exacerbation of asthma. Which of the following features is most likely to indicate a life-threatening attack? A. Failure to improve after nebulized salbutamol 5 mg B. Oxygen saturations of 94% on room air C. Peak flow of 30% best or predicted D. Respiratory rate of 28/min Answer: C 0026. A 26-year-old man is admitted with acute severe asthma. Treatment is initiated with 100% oxygen, nebulized salbutamol, ipratropium bromide nebulizers, and IV hydrocortisone. Despite initial treatment, there is no improvement. Examination reveals that he is in respiratory distress, cannot complete full sentences, and has bilateral expiratory wheezing. Which of the following is the most appropriate IV emergency treatment? A. Magnesium sulfate B. Aminophylline C. Salbutamol D. Adrenaline Answer: A 0027. A 70-year-old man known to have chronic obstructive pulmonary disease (COPD). Examination revealed evidence of cor pulmonale with a significant degree of lower limb edema. His FEV1 is 40% (see lab result). Which of the following interventions is most associated with increased survival? A. Heart-lung transplant B. Corticosteroid therapy C. Pulmonary rehabilitation D. Long-term oxygen therapy Answer: D 0028. A 55-year-old man is evaluated for an 18-month history of intermittent, nonproductive chronic cough, and dyspnea with exertion. No history of fever, chest pain, heartburn, loss of appetite, or weight loss. He has smoked 1 pack daily for the last 30 years. His medical history is otherwise unremarkable, and he takes no medications. Examination revealed he is comfortable, no jugular venous distention, normal heart sounds, no murmur, clear lungs, and no lower limb edema. A chest radiograph and electrocardiogram are normal. Which of the following is the most appropriate next diagnostic test? A. Sleep study B. Spirometry C. Chest CT scan D. Bronchoscopy Answer: B 0029. A 79-year-old man is evaluated in follow-up for a 2 mm solitary pulmonary nodule, which was incidentally discovered on imaging for chest pain 4 years ago. Follow-up chest CT was obtained at 1, 2, and 3-year intervals, and the pulmonary nodule appears unchanged. He feels well and does not have any respiratory or constitutional symptoms. He never smokes. Examination revealed normal vital signs and cardiopulmonary examination. Which of the following is the most appropriate recommendation? A. Repeat chest CT in 1 year B. Repeat chest CT in 2 years C. Repeat chest CT in 5 years D. No further imaging is necessary Answer: D 0030. A 68-year-old woman is evaluated in follow-up after a 5 mm pulmonary nodule was discovered on chest imaging obtained 9 months ago. She feels well and has not had shortness of breath, cough, hemoptysis, weight loss, or chest pain. She has a 50-pack-year smoking history and is a recurrent smoker. Examination revealed normal vital signs, no cervical or supraclavicular lymphadenopathy, and clear lungs to auscultation bilaterally (see report). Repeated chest CT: Interval increase in the size of the nodule, now measuring 10.5 mm in diameter, and no evidence of mediastinal or hilar lymphadenopathy. Which of the following is the most appropriate next step? A. Perform blind transthoracic needle aspiration B. Repeat chest CT imaging in 6 months C. Refer for thoracic surgery D. Perform bronchoscopy Answer: C 0031. A 69-year-old man complains of shortness of breath and weight loss for the last 6 months. He has a 40-pack-year smoking history and no TB or HIV risk factors (see reports). Chest X-ray: A large left-sided pleural effusion. Thoracentesis: 1,500 mL of bloody fluid that is strongly exudative by Light's criteria. Which of the following is the most appropriate next step? A. Empirical broad-spectrum antibiotics B. CT chest with contrast C. Blind pleural biopsy Answer: B 0032. A 55-year-old woman is referred to the clinic because of a 1 cm solitary nodule with sharp borders in the right lung. The nodule was detected on an abdominal CT performed to evaluate abdominal pain. She does not have any respiratory complaints. She smoked 20 cigarettes per day for 10 years but managed to stop smoking 20 years ago. Which of the following is the most appropriate next step? A. Follow-up CT in 3-6 months B. No follow-up needed C. Transbronchial biopsy D. Surgical resection Answer: A 0033. A 71-year-old man is admitted to the hospital because of high fever and dyspnea associated with purulent sputum. Physical examination reveals dullness on percussion on the right lower chest and rales on auscultation (see reports). Chest radiography: Pneumonic infiltrate in the right upper lobe and a pleural effusion. Aspiration: Pending result. Which of the following pleural fluid analyses will most likely indicate the need for chest-tube drainage? A. pH < 7.2 B. Glucose > 60 mmol/L C. Negative Gram stain D. Serosanguineous appearance Answer: A 0034. A 29-year-old woman in the second trimester of pregnancy presents to the Emergency Room due to progressive dyspnea for the past 24 hours. Examination reveals clear lungs and a swollen right lower limb (see lab results and report). Chest radiography: Normal. Which of the following is the most important initial diagnostic procedure? A. D-dimer B. CT pulmonary angiography C. Lung perfusion scintigraphy D. Doppler ultrasonography of the legs Answer: D 0035. A 50-year-old man is 6 days post-uneventful subtotal colectomy and ileocolic anastomosis when he develops sudden-onset shortness of breath and one episode of hemoptysis. Examination reveals mildly decreased breath sounds at the left lung base, normal S1 and S2, a soft and lax abdomen with normal bowel sounds, and no lower extremities edema or tenderness (see lab results and reports). Chest X-ray: Left basilar atelectasis. ECG: Sinus tachycardia and a new finding of a right bundle branch block. Which of the following is the most appropriate test? A. D-dimer level B. Echocardiography C. CT pulmonary angiography D. Lower limb Doppler ultrasound Answer: C 0036. A 30-year-old man presents with cough, fever, and right-sided pleuritic chest pain for 2 days. He is a non-smoker and works as a school teacher. He has no significant past medical history and does not take any medicine. Examination reveals bronchial breath sounds at the right lower chest (see report). Chest X-ray: Right lower lobe pneumonia and 25 mm pleural effusion on decubitus film. Which of the following is the most appropriate next step in the management? A. Perform diagnostic thoracentesis B. Start anti-tuberculosis treatment C. Perform bronchoalveolar lavage D. Intravenous antibiotics Answer: A 0037. A 44-year-old woman is seen in the clinic with difficult-to-control asthma despite being on high-dose inhaled steroids and frequent courses of systemic steroids. She also has a history of allergic rhinitis and nasal polyps. She claims to use aspirin and ibuprofen for migraine almost daily. She is a non-smoker and works in a school. Examination reveals nasal polyps and bilateral expiratory wheezing on chest auscultation (see report). Chest X-ray: Normal. Which of the following is the most appropriate next step in management? A. Chest CT scan B. Direct laryngoscopy to assess upper airways C. Blood eosinophil count and serum IgE level D. Discontinue aspirin and ibuprofen and start Montelukast Answer: D 0038. A 63-year-old man is evaluated for hemoptysis and a 5-kg weight loss over the last 2 months. He has a history of 30 pack-years of smoking but stopped 10 days ago (see reports). Chest X-ray: 5-cm irregular mass in the right upper lobe. Fiberoptic bronchoscopy: Tumor in the right upper lobe bronchus. Biopsy: Squamous cell carcinoma. Whole body FDG-PET scan: Negative for metastasis. Which of the following is the most appropriate action while preparing him for surgery? A. Repeat PET scan in 3 months B. Pulmonary function tests C. Chemotherapy D. Radiotherapy Answer: B 0039. A 55-year-old man with a long history of smoking presents to the Emergency Room with exertional shortness of breath and a daily productive cough of 3-4 tablespoons of yellow sputum that has worsened over the last 3 days. No history of fever, chest pain, or hemoptysis. Chest examination reveals decreased breath sounds and scattered rhonchi. Which of the following is the most appropriate first-line inhaling agent to be started in an emergency? A. Ipratropium bromide B. Beclomethasone C. Salmeterol D. Albuterol Answer: A 0040. A 33-year-old woman with rheumatoid arthritis for 3 years is evaluated in the Emergency Department with a 3-day history of pain and swelling of the left knee and low-grade fever. There was no history of trauma. Her medications are prednisone, methotrexate, and etanercept. Joints examination reveals a swollen and tender left knee, and mild synovitis is present on the third proximal interphalangeal joint bilaterally. Which of the following is the most appropriate initial management? A. Bone scan B. Arthrocentesis of the left knee C. Conventional radiology of the left knee D. Magnetic resonance imaging (MRI) of the left knee Answer: B 0041. A 40-year-old woman presents with pain and swelling of the hands with early morning stiffness for 3 months. She has photosensitivity, hair loss, difficulty climbing stairs, and Raynaud's phenomenon. Examination reveals active arthritis at the second and third proximal interphalangeal joints, malar rash, and healed ulcers at the tip of the second finger. Proximal muscle strength is 3/5 (see lab results). Antinuclear antibody (ANA titer 1: 2540) Which of the following antibodies will most likely confirm the diagnosis? A. Anti-Smith (SM) B. Anti-ribonucleoprotein (RNP) C. Anti-double stranded DNA (anti-ds DNA) D. Anti-cyclic citrullinated peptide (anti-CCP) Answer: B 0042. A 28-year-old woman with active neuropsychiatric lupus based on clinical manifestations and her workups. Which of the following is the most appropriate first-line therapy? A. Corticosteroids and cyclophosphamide B. Intravenous cyclophosphamide C. Corticosteroids and rituximab D. Intravenous corticosteroids Answer: A 0043. A 35-year-old man is evaluated during a follow-up visit. He is a known case of rheumatoid arthritis that is well controlled with prednisone 15 mg and hydroxychloroquine 400 mg daily. His physical examinations are unremarkable (see lab results). Which of the following is the most appropriate management? A. Taper prednisone and start methotrexate B. Taper prednisone and start ibuprofen C. Taper prednisone and start cyclosporine D. Continue his treatment Answer: A 0044. A 50-year-old man is evaluated in the clinic for newly diagnosed hypertension that is not well controlled with lifestyle modifications. His systemic examinations are unremarkable (see lab results). Which of the following is the most appropriate management for this patient's hypertension? A. Amlodipine B. Lisinopril C. Hydrochlorothiazide D. Lisinopril and amlodipine Answer: D 0045. A 55-year-old man with acute coronary syndrome who recently underwent cardiac catheterization is evaluated because of acute kidney injury. Which of the following is most suggestive of cholesterol emboli as a cause of acute renal failure? A. White blood cell casts B. Red cell casts C. Granular casts D. Eosinophiluria Answer: D 0046. A 60-year-old man who has been on hemodialysis for the last 10 years is evaluated in the clinic. He is complaining of fatigue but has no other symptoms. His physical examination is unremarkable (see lab results). Which of the following is the most common cause of death in such patients? A. Cardiovascular disease B. Hyperkalemia C. Anemia D. Uremia Answer: A 0047. A 30-year-old woman with Sjogren's syndrome on hydroxychloroquine is evaluated in the clinic because of fatigue. Her physical examinations are unremarkable (see lab results). Which of the following renal tubular acidosis type is the most likely cause of her hypokalemia? A. Type 1 B. Type 2 C. Type 3 D. Type 4 Answer: A 0048. A 35-year-old woman presents to the Emergency Department with fever and shortness of breath for 2 days. She is a known rheumatoid arthritis patient on hydroxychloroquine 400 mg daily. Examination reveals crepitations at the right lower lung. Renal and liver function are normal. She was treated with cefuroxime with resultant significant improvement. However, her renal function is deteriorating (see lab results). Which of the following is the most likely cause of kidney function deterioration? A. Renal vasculitis B. Acute interstitial nephritis C. Prerenal acute kidney failure D. Immune complex glomerulonephritis Answer: B 0049. An 18-year-old man is evaluated because of acute kidney injury 3 days after an upper respiratory tract infection. Examination reveals lower limb pitting edema up to the knee levels. His laboratory investigations were normal 3 days ago but are repeated today (see lab results). Which of the following is the most likely diagnosis? A. IgA nephropathy B. Lupus nephritis C. Acute interstitial nephritis D. Post-infectious glomerulonephritis Answer: A 0050. A 28-year-old man who recently joined military service and underwent excessive military training presents to the Emergency Department with lightheadedness on standing and fatigue. Physical examination reveals postural hypotension, otherwise unremarkable. Urine osmolality is elevated > 500 mosmol/L (see lab results). What is the most likely etiology for his acute kidney injury? A. Pre-renal azotemia B. Acute interstitial nephritis C. Acute glomerulonephritis D. Acute tubular necrosis (ATN) Answer: A 0051. A 28-year-old man who recently joined military service and underwent excessive military training presents to the Emergency Department with lightheadedness on standing, muscle cramping, and fatigue. Physical examination reveals postural hypotension, otherwise unremarkable. A diagnosis of acute kidney injury due to hypovolemia is established. Which of the following is the most expected laboratory abnormality? A. Low urine osmolarity < 250 mosmol/L B. Fractional excretion of sodium > 2% C. Blood urea nitrogen/creatinine ratio is elevated > 20 D. Presence of dysmorphic red blood cell (RBC) and RBC cast Answer: C 0052. A 55-year-old man with stage 2 chronic kidney disease is evaluated because of anemia. He is complaining of fatigue. His physical examination reveals pale skin, otherwise unremarkable (see lab results). Which of the following is the most appropriate investigation to be done? A. Serum iron studies B. Reticulocyte count C. Serum erythropoietin level D. Bone marrow aspiration Answer: A 0053. A 64-year-old man known to have liver cirrhosis secondary to chronic hepatitis C completed treatment for HCV and now has undetectable HCV RNA 12 weeks post-treatment. He is asymptomatic. Clinically, he has a few spider angiomata and mild splenomegaly (see lab results). Which of the following is the most appropriate follow-up option? A. MRI abdomen yearly B. Reassurance and follow-up C. Liver ultrasound every 6 months D. Alpha fetoprotein every 3 months Answer: C 0054. A 34-year-old woman, a known case of SLE on hydroxychloroquine, came to the clinic for a routine checkup. She is asymptomatic with an unremarkable clinical examination. Urine analysis and her blood tests are done. She underwent renal biopsy, which revealed Class I (minimal change) lupus nephritis (see lab results). Which of the following is the most appropriate initial management? A. Reassurance B. Pulse steroid C. High-protein diet D. Control of blood pressure Answer: D 0055. A 26-year-old man is evaluated in the Emergency Department 1 hour after having a first- time generalized tonic-clonic seizure. He has never had a seizure previously or any episode of jerking, staring, confusion, or memory loss. He has had no recent illness and has no history of neurological problems. Birth and development were normal. There is no family history of seizures or epilepsy. He takes no medication and does not use illicit drugs. His vital signs are normal. Neurological findings are unremarkable. Results of laboratory studies show a normal complete blood count and metabolic panel. Which of the following is the most appropriate initial step in management? A. ECG B. EEG C. Head CT scan D. Lumbar puncture Answer: C 0056. A 44-year-old man is presented to the Emergency Room 8 hours after the sudden onset of a global, severe headache with associated neck stiffness. The headache is described as the strongest he has ever had. He is known to have hypertension but is noncompliant with his medications. Examination revealed a discomfortable and restless patient. Funduscopic examination is normal with the left pupil being 2 millimeters larger than the right and poorly reactive to light, and he has neck stiffness with passive movement (see report). Which of the following is the most appropriate next diagnostic test? A. Magnetic resonance angiography of the neck B. No further testing is necessary C. Lumbar puncture D. MRI of the brain Answer: C 0057. A 55-year-old man is evaluated 2 days after having an episode of left arm weakness without pain that lasted 5 minutes. He is now asymptomatic. He has type 2 diabetes mellitus and dyslipidemia. Medications are aspirin, insulin, and atorvastatin. Physical examination shows that no carotid bruits or cardiac murmurs are heard on cardiac auscultation, and all other physical examination findings are normal (see report). Electrocardiogram: Normal sinus rhythm with no ST-segment or T-wave changes. Which of the following is the most appropriate initial imaging test? A. Transesophageal echocardiography B. Carotid duplex ultrasonography C. CT angiography of the neck D. MRI of the brain Answer: B 0058. A 59-year-old man is evaluated in the hospital for progressive shortness of breath and hypoxia 20 hours after undergoing surgery for colonic adenocarcinoma with known liver metastases (see report). CT angiography: Shows a right lower lobe pulmonary embolism. Which of the following is the most appropriate initial treatment? A. Warfarin B. Enoxaparin C. Rivaroxaban D. Fondaparinux Answer: B 0059. A 30-year-old woman is evaluated for new-onset nonproductive cough and dyspnea on exertion. She is pregnant at 30 weeks gestation. Medical history is unremarkable. Her only medication is a prenatal vitamin. Cardiopulmonary examination is normal. She has a gravid uterus and 2+ edema of the lower extremities without calf tenderness. Doppler ultrasonography of both legs is negative for deep venous thrombosis (see lab results). Which of the following is the most appropriate diagnostic test to perform next? A. D-dimer assay B. CT angiography C. Pulmonary function testing D. Ventilation-perfusion lung scan Answer: D 0060. A 49-year-old woman undergoes perioperative evaluation. She has osteoarthritis of the right hip and is scheduled for elective hip arthroplasty. Medical history is otherwise notable for type 2 diabetes mellitus. Her menstrual period has been heavy for the last few months. Medications are ibuprofen and metformin. Vital signs are normal. She has painful and limited range of motion in the right hip (see lab results). Which of the following is the most appropriate test to perform next? A. Iron studies B. Vitamin B12 level C. No further evaluation D. Hemoglobin electrophoresis Answer: A 0061. A 29-year-old woman arrives at the hospital in labor at 38 weeks gestation of an otherwise uncomplicated second pregnancy. Her labor fails to progress after 20 hours; a cesarean section delivers a healthy female infant. A few minutes after the procedure, the patient begins to have heavy vaginal bleeding. Placental abruption is diagnosed and appropriately managed with fluid resuscitation and transfusion of 4 units of blood, yet she continues to have moderate bleeding (see lab results). Which of the following infusion is the most appropriate management? A. Cryoprecipitate B. Platelets C. Albumin D. Iron Answer: A 0062. A 70-year-old woman presents with fatigue and exertional dyspnea for several months. Medical history includes longstanding hypertension and chronic kidney disease, and she is not adherent to her medications. Examination reveals pale mucous membranes and nail beds (see lab results and report). Kidney ultrasonography: Showed small kidneys bilaterally with echographic features suggesting chronic kidney disease. Which of the following is the most likely cause of this patient's anemia? A. Erythropoietin deficiency B. Myelodysplastic syndrome C. Iron deficiency D. Inflammation Answer: A 0063. A 32-year-old woman presents to the clinic with decreased exercise tolerance and ice cravings for the past several months. Medical history is notable for celiac disease diagnosed 7 years ago, and she is not compliant with a gluten-free diet. Her only medication is ferrous sulfate tablets 325 mg once daily, which she has been taking for 12 weeks after being diagnosed with iron deficiency anemia. Examination reveals a thin patient with normal vital signs, pale conjunctivae, and nail beds. Cardiac examination reveals a grade 2/6 systolic flow murmur (see lab results). Which of the following is the most appropriate treatment? A. Continue the same treatment B. Intravenous iron preparation C. Oral iron in a liquid preparation D. Oral iron tablets three times daily Answer: B 0064. A 58-year-old woman presents to the follow-up clinic. She is known to have idiopathic deep venous thrombosis and is on warfarin, which was initiated 6 weeks ago. The examination is unremarkable (see lab result). Which of the following is the most appropriate management in addition to withholding warfarin? A. Administer vitamin K B. Re-measure INR in 2 days C. Administer fresh frozen plasma D. Administer prothrombin complex concentrate Answer: B 0065. A 34-year-old woman at 38 weeks gestation is evaluated for a severe headache. Her only medication is a prenatal vitamin. Neurologic and cardiopulmonary examination is normal. She has a gravid uterus and 2+ edema of the lower extremities. No petechiae or ecchymoses are seen (see lab results and report). Peripheral smear: Occasional fragmented erythrocytes without platelet clumping. Which of the following is the most appropriate management? A. High-dose dexamethasone B. Emergent delivery C. Plasma exchange D. Eculizumab Answer: B 0066. A 31-year-old woman known to have rheumatic mitral stenosis recently developed atrial fibrillation, for which she was placed on warfarin therapy. What is the most appropriate target of the international normalized ratio (INR) range? A. 3.0 Answer: C 0067. A 25-year-old woman is seen for a routine prenatal evaluation; she is 12 weeks pregnant. This is her second pregnancy; the first pregnancy was uncomplicated. Medical history is notable for sickle cell disease requiring 1 to 2 hospitalizations per year for painful events. She has no history of stroke or acute chest syndrome. Her only medication is a folic acid supplement. Cardiac examination reveals a grade 2/6 systolic flow murmur (see lab results). Which of the following is the most appropriate treatment? A. Exchange transfusion throughout pregnancy B. Close observation throughout pregnancy C. Simple blood transfusion D. Start on hydroxyurea Answer: B 0068. A 55-year-old woman with type 2 diabetes mellitus was recently diagnosed with hypertension. Which of the following is the most appropriate antihypertensive medication? A. Hydralazine B. Amlodipine C. Metoprolol D. Ramipril Answer: D 0069. A 60-year-old man is found to have elevated blood pressure during a routine checkup. He has no symptoms, and physical examination is unremarkable. ECG, CBC, electrolytes, renal function tests, and urine analysis are normal. Which of the following is the most appropriate next option in management? A. Request echocardiogram B. Start antihypertensive medication C. Perform ambulatory blood pressure measurement D. Repeat office blood pressure measurement in 3 months Answer: C 0070. A 75-year-old man with COPD presented with acute exacerbation due to super-added pneumonia. He was managed with IV fluids, antibiotics, nebulized salbutamol, and nasal oxygen at 4 L/min. A few hours later, he started to become drowsy (see lab results). Which of the following is the most appropriate immediate management? A. Noninvasive ventilation B. Nebulized Ipratropium C. Reduce inspired oxygen concentration D. Increase the frequency of nebulized salbutamol Answer: C 0071. A 55-year-old man presented with a productive cough for 3 months. He had a similar attack of chronic cough for the previous 2 years. He smokes 1 pack/day for the last 15 years. Examination revealed an obese patient with scattered wheeze and crackles over the lungs. Which of the following is the most likely diagnosis? A. Bronchiectasis B. Bronchial asthma C. Pulmonary fibrosis D. Chronic bronchitis Answer: D 0072. A 55-year-old man, a plumber, presented with a 1-year history of dry cough and progressive exertional breathlessness. Examination shows finger clubbing and bilateral basal crackles (see reports). Chest X-ray: Bilateral reticular shadowing at the lower zone of the lungs. Which of the following has the highest diagnostic value? A. Trans-Bronchial Biopsy B. Measures diffusion capacity on PFT C. High-resolution CT scan of the chest D. Serum angiotensin-converting enzyme Answer: C 0073. A 65-year-old man known to have COPD admitted with 2 weeks of worsening productive cough and breathlessness. Examination shows that he is drowsy with bilateral wheeze over the chest (see lab results). Which of the following is the most appropriate next step in management? A. High flow oxygen B. Inhaled albuterol C. Intravenous aminophylline D. Nebulized salbutamol and ipratropium Answer: D 0074. A 61-year-old obese man presented with daytime somnolence, poor concentration, nocturia, and progressive shortness of breath. Examination reveals an obese patient with bilateral lower limb edema and normal vesicular breath sounds on lung auscultation. Which of the following is the most appropriate investigation to establish the diagnosis? A. Chest X-ray B. Sleep study C. Echocardiography D. High-resolution CT lung Answer: B 0075. A 62-year-old diabetic woman was admitted to the Intensive Care Unit with pneumonia and sepsis (see lab results and report). Chest X-ray: Right lobe consolidation and pleural effusion. Which of the following most likely indicates a poor prognosis? A. Age B. Diabetes mellitus C. High white blood cell count D. Pleural effusion on chest X-ray Answer: B 0076. A 64-year-old diabetic woman was admitted to the Intensive Care Unit with pneumonia and sepsis. She has no chronic lung disease and is not a smoker (see lab results and report). Chest X-ray: Right lobe consolidation and pleural effusion. Which of the following is the most appropriate empiric antibiotic? A. Moxifloxacin B. Azithromycin C. Vancomycin D. Ceftazidime Answer: A 0077. A 65-year-old woman presented with a 4-month history of progressive shortness of breath, dry cough, and fatigue. She was treated for pneumonia with 2 courses of antibiotics but showed no improvement in her symptoms. She has no significant medical problems (see reports). Chest X-ray (after antibiotics): A persistent dense area of consolidation at the right lower lobe. Bronchioalveolar lavage: Atypical cells. Which of the following is the most likely diagnosis? A. Sarcoidosis B. Atypical pneumonia C. Allergic pneumonitis D. Bronchogenic cancer Answer: D 0078. A 45-year-old woman, a smoker for 18 years, presented with exertional shortness of breath and a chronic cough for 5 months. She is also known to have long-standing rheumatoid arthritis. Chest examination revealed increased AP diameter with hyperresonance all over the chest and scattered crackles (see report). Pulmonary function test: Increase in total lung capacity and residual volume. FEV1 60% with no change post-bronchodilator. Which of the following is the most likely diagnosis? A. Lung fibrosis B. Emphysema C. Bronchiectasis D. Allergic pneumonitis Answer: B 0079. A 17-year-old student presented with recurrent attacks of mild wheeze and shortness of breath despite using inhaled salbutamol regularly. Which of the following is the best next step in the management? A. Oral prednisolone B. Inhaled corticosteroid C. Low-dose aminophylline D. Inhaled ipratropium bromide Answer: B 0080. A 70-year-old man presented to the Emergency Department with a 2-hour history of confusion and irritability. He was well until 5 days ago, when he injured his right leg and has been bed-bound since the accident. He is hypertensive and well-controlled on medication. He has no fever (see reports). Chest X-ray: Normal. ECG: Sinus tachycardia. Which of the following is the most likely diagnosis? A. Arrhythmia B. Cerebral infarction C. Brain hemorrhage D. Pulmonary embolism Answer: D 0081. A 65-year-old man, a smoker who was recently diagnosed with COPD, complains of shortness of breath on moderate exertion. He has no other medical problems, and chest examination was unremarkable. Which of the following is the most appropriate initial step in management? A. Inhaled steroid B. Reduced activity C. Smoking cessation D. Oral ipratropium Answer: C 0082. A young man with HIV was admitted to the Intensive Care Unit with progressive shortness of breath, cough, and fever. Examination showed he was dyspneic and cyanosed (see report). Chest X-ray: Signs of ARDS. Which of the following is the most likely causative organism? A. Staphylococcus aureus B. Pneumocystis jiroveci C. Pseudomonas aeruginosa D. Streptococcus pneumoniae Answer: B 0083. A 17-year-old man developed febrile neutropenia and shock post-chemotherapy for lymphoma. Which of the following is the most appropriate initial antibiotic? A. Meropenem B. Vancomycin C. Cefuroxime D. Caspofungin Answer: A 0084. A 45-year-old man was admitted to the Intensive Care Unit with fever. He required a vasopressor to maintain a mean arterial pressure (MAP) of more than 65 mm Hg. He is receiving adequate IV fluid resuscitation (see lab result). Which of the following is the most likely diagnosis? A. Sepsis B. Septicemia C. Septic shock D. Systemic inflammatory response syndrome Answer: C 0085. A 35-year-old woman presented to the clinic with a 3-month history of fatigue, lethargy, and weight gain. Physical examination is unremarkable (see lab results). Which of the following is the most appropriate investigation? A. Thyroid-stimulating hormone (TSH) B. Thyroid peroxidase antibody (TPO) C. Triiodothyronine (T3) D. Thyroxine (T4) Answer: A 0086. A 32-year-old woman is evaluated because of a 3-month history of weight loss, heat intolerance, tremor, and irritability. Physical examination reveals a tremor and a non-tender goiter. She has negative thyroid-stimulating immunoglobulins (see lab results). Which of the following is the most likely diagnosis? A. Primary TSH-secreting tumor B. Tertiary hypothyroidism C. Toxic nodular goiter D. Grave's disease Answer: C 0087. A 25-year-old man with type 1 diabetes mellitus on insulin therapy was found in the clinic waiting area drowsy and excessively sweating. A quick measurement of blood glucose revealed a low level. Which of the following is the most appropriate action to be taken? A. Give fruit juice B. Start IV 0.9% normal saline C. Glucagon subcutaneously D. IV 5% dextrose with normal saline Answer: A 0088. A 32-year-old housemaid was brought to the Emergency Room with headache, confusion, and fever for 2 weeks. She just came from India 1 month ago. Examination reveals a confused patient with neck rigidity. Fundus examination shows papilledema (see lab results and report). Brain CT scan: No masses or hemorrhage. Which of the following is the most appropriate next step in management? A. Treat household contacts against TB B. Admit the patient to a negative pressure isolation room C. Send CSF fluid for TB culture and wait for the results D. Start isoniazid (INH), rifampin, pyrazinamide, and ethambutol Answer: D 0089. A 27-year-old woman presents to the clinic with a 1-week history of fever, sore throat, difficulty swallowing, and painful swelling in her neck. She had an upper respiratory tract infection 3 weeks ago. She has no past medical history. Examination reveals a tender goiter. Her oropharynx is clear (see lab results). Which of the following is the most likely diagnosis? A. Sick euthyroid syndrome B. Subclinical hypothyroidism C. Hashimoto's thyroiditis D. Subacute thyroiditis Answer: D 0090. A 44-year-old Saudi man is evaluated because of a 3-month history of fever and back pain that radiates to his legs. His examination reveals bony tenderness over the entire back. Blood culture shows gram-negative coccobacilli. Which of the following is the most likely diagnosis? A. Brucellosis B. Tuberculosis C. Leishmaniasis D. Schistosomiasis Answer: A 0091. A 25-year-old man was recently diagnosed with inflammatory bowel disease. Which of the following features best distinguishes Crohn's disease from ulcerative colitis? A. Aphthous ulcers B. Colonic involvement on colonoscopy C. Non-caseating granuloma in biopsy D. Presence of extra-intestinal manifestations Answer: C 0092. A 35-year-old woman presents to the clinic with a 5-month history of fatigue. She takes no medication. Her examination is unremarkable (see lab results). What is the most appropriate treatment for this patient's dyslipidemia? A. Statin B. Fibrate C. Niacin D. Thyroxine Answer: D 0093. A 38-year-old man with a 10-year history of gastroesophageal reflux disease on pantoprazole is evaluated in the clinic because of a 4-month history of intermittent dysphagia for solids. What is the most appropriate next investigation? A. Endoscopy B. Manometry C. Barium swallow D. Fecal occult blood test (FOB) Answer: A 0094. A 60-year-old man is evaluated in the clinic because of a 6-month history of exertional shortness of breath. Clinical examination reveals a positive Corrigan's pulse, early diastolic murmur at the left upper sternal border, and bilateral fine basal crepitations (see report). Echocardiogram: Severe aortic regurgitation and ejection fraction of 40%. Which of the following is the most appropriate treatment? A. Diuretics B. Calcium channel blockers C. Surgical valve replacement (AVR) D. Angiotensin-converting enzyme (ACE) inhibitors Answer: C 0095. A 45-year-old man with diabetes mellitus for 4 years on metformin is evaluated in the clinic because of an abnormal liver function test. He is clinically asymptomatic. His examination is unremarkable (see lab results). Which of the following is the most likely cause of his abnormal liver function test? A. Acute viral hepatitis B. Hemochromatosis C. Autoimmune hepatitis D. Non-alcoholic fatty liver disease Answer: D 0096. A 55-year-old man with liver cirrhosis on regular follow-up presents to the Emergency Room with a 2-day history of decreased level of consciousness and increased abdominal girth. Examination reveals an ill and cachectic patient with tense ascites and lower limb pitting edema bilaterally. Which of the following is the most appropriate management? A. Loop diuretics B. Start diuretic and spironolactone C. Transjugular intrahepatic portosystemic shunt (TIPS) Answer: B 0097. A 50-year-old man with cirrhosis presented to the Emergency Room with increasing ascites and decreased level of consciousness over one week. Examination revealed a confused patient with tense ascites (see lab results). Which of the following is the most appropriate modified Child-Turcotte-Pugh score of this patient? A. Stage A B. Stage B C. Stage C D. Not applicable in cirrhosis Answer: B 0098. A 55-year-old man with diabetes mellitus for 10 years was admitted to the Intensive Care Unit with severe pneumonia that required intubation and ventilation. He was treated with intravenous antibiotics, and his vital signs, including his blood pressure, were maintained within the normal range. However, last night, he developed hypotension that required two inotrope agents (see lab results). Which of the following is the most likely diagnosis? A. Acute cholangitis B. Ischemic hepatitis C. Acute viral hepatitis D. Drug-induced hepatitis Answer: B 0099. A 23-year-old man presents to the clinic with a 7-month history of recurrent episodes of abdominal pain and diarrhea that are sometimes associated with streaks of blood and occasional audible peristalsis. He had fever and loss of appetite for the last 7 days. There is no history of allergy or chronic use of any medications. Examination reveals a thin, ill-looking patient with a tender but soft abdomen (see lab results). Which of the following is the most likely diagnosis? A. Celiac disease B. Lactose intolerance C. Irritable bowel syndrome D. Inflammatory bowel disease Answer: D 0100. A 50-year-old man known to have chronic viral hepatitis presented with ascites and was found to have a high serum ascites albumin gradient (SAAG) of more than 1.1. Which of the following is the most likely cause? A. Pancreatitis B. Tuberculosis C. Liver cirrhosis D. Nephrotic syndrome Answer: C 0101. A 35-year-old Indian man presents to the Emergency Department with lethargy, weight loss, and headache for 3 months. He has a history of contact with a tuberculosis patient several years ago. His neurological and ophthalmologic examination are unremarkable. He has a positive polymerase chain reaction for Mycobacterium tuberculosis (see lab results). Which of the following is the most appropriate treatment? A. Isoniazid, rifampin, pyrazinamide, ethambutol B. Isoniazid, rifampin, pyrazinamide, ethambutol and diuretics C. Isoniazid, rifampin, pyrazinamide, ethambutol and acyclovir D. Isoniazid, rifampin, pyrazinamide, ethambutol and dexamethasone Answer: D 0102. A 29-year-old nurse is evaluated in the clinic for employment contract renewal. She is asymptomatic, and her clinical examination is unremarkable. Her laboratory tests are within normal range. Her skin purified protein derivative (PPD) test after 48 hours is 12 mm. Which of the following is the most appropriate management? A. No intervention is required B. Isoniazid 300 mg daily for 3 months C. Isoniazid 300 mg daily for 6 months D. Isoniazid, rifampin, and ethambutol for 6 months Answer: C 0103. A 48-year-old woman is evaluated for persistent fever and flank pain. She was admitted several days ago for treatment of pyelonephritis. Her fever is persistent after 5 days of starting an appropriate antibiotic. Examination reveals tenderness over the left loin area, otherwise unremarkable (see report). Urine culture: Positive for Escherichia coli susceptible to the antibiotic that was already started. Which of the following is the most appropriate management? A. Kidney ultrasound B. Change the antibiotic C. Repeat urine culture D. Continue observation Answer: A 0104. A 33-year-old pregnant woman in her third trimester is evaluated in the clinic for her routine checkup visit. She is asymptomatic and denies a history of fever, dysuria, or urgency. Her examination is unremarkable (see lab results and report). Urine culture: Positive for Escherichia coli susceptible to different broad-spectrum antibiotics. Which of the following is the most appropriate treatment? A. Amoxicillin B. Gentamicin C. Observation D. Nitrofurantoin Answer: A 0105. A 35-year-old man presents to the clinic with a 4-month history of fever, night sweats, fatigue, arthralgia, and lower back pain. He had a history of unpasteurized milk ingestion. His examination reveals right knee arthritis and limited spinal movements. His cardiopulmonary examination is unremarkable (see lab results and reports). Right knee joint aspiration: WBC 15,000 (0-200). Blood culture: Negative. Which of the following is the most appropriate next action? A. Liver biopsy B. Bone marrow biopsy C. Repeat blood culture D. Serology for brucellosis Answer: D 0106. A 33-year-old woman was admitted to the hospital after a road traffic accident that resulted in multiple fractures and had undergone multiple orthopedic surgical procedures. She has a positive urinary culture for Candida albicans. She denies any history of fever or urinary symptoms. She had an indwelling urinary catheter. Her examination is unremarkable. Which of the following is the most appropriate management? A. Fluconazole B. Caspofungin C. Amphotericin B D. No antifungal therapy Answer: D 0107. A 22-year-old man is evaluated for a 2-day history of dysuria and urethral discharge. He had recent sexual activity with a new female partner. He is afebrile, and his examination is unremarkable (see report). Which of the following is the most appropriate treatment? A. Gentamycin B. Azithromycin C. Ceftriaxone D. Nitrofurantoin Answer: B 0108. A young woman with acute uncomplicated pyelonephritis is evaluated in the Emergency Department. Which of the following is the most appropriate management? A. Treat as outpatient with ciprofloxacin B. Treat as outpatient with clarithromycin C. Admit and start carbapenem antibiotics D. Admit to start ceftriaxone and vancomycin Answer: A 0109. A 45-year-old man presents to the Emergency Department with a 3-day history of fever and a productive cough of yellowish sputum. Medical history is significant for type 2 diabetes mellitus. He is a smoker. His chest examination reveals decreased breath sounds throughout and crackles at the right lung base. Other systemic examination is unremarkable (see report). Chest radiograph: Right lower lobe infiltrate. Which of the following is the most appropriate management? A. Admit him, start ceftriaxone and azithromycin B. Admit him, start intravenous amoxicillin C. Outpatient treatment with azithromycin D. Outpatient treatment with cefuroxime and azithromycin Answer: D 0110. A patient with highly suspicious pulmonary tuberculosis is evaluated in the Emergency Department. Standard universal precautions are taken. Which of the following is the most appropriate infection control precaution during the assessment? A. Standard precautions B. Standard and droplet precautions C. Standard and contact precautions D. Standard and airborne precautions Answer: D 0111. A young man who is sexually active is evaluated in the clinic for clinical features that suggest gonorrhea. Which of the following tests is the most appropriate to establish the diagnosis? A. Blood culture B. Urinary culture C. Urethral swabs D. Vesicular rash swabs Answer: C 0112. A 60-year-old man who has a history of mitral valve replacement with a mechanical prosthesis is admitted with infective endocarditis (see report). Blood culture: Positive for methicillin-resistant Staphylococcus aureus (MRSA). Which of the following is the most appropriate antibiotic regimen? A. Vancomycin and rifampin B. Vancomycin and gentamicin C. Vancomycin and ceftriaxone D. Vancomycin, rifampin, and gentamicin Answer: D 0113. A 60-year-old man with diabetes and hypertension presented to the Emergency Department with 3-hour continuous retrosternal heaviness associated with profuse sweating. Physical examination reveals tachycardia and an S4 gallop on heart auscultation. The patient is commenced on aspirin, clopidogrel, metoprolol, and nitroglycerin infusion (see reports). Electrocardiogram: Sinus tachycardia with left bundle branch block. Previous ECG was normal. Cardiac enzymes: Pending. Which of the following is the most appropriate next step in management? A. Start lidocaine infusion B. Perform exercise ECG testing C. Administer thrombolytic therapy D. Wait for the results of cardiac enzymes Answer: C 0114. A 44-year-old man complains of dyspnea on exertion. Heart auscultation reveals an early diastolic murmur at the base of the heart. The intensity and duration of the murmur increase during inspiration. Which of the following is the most likely cause? A. Mitral regurgitation B. Aortic regurgitation C. Tricuspid regurgitation D. Pulmonary regurgitation Answer: D 0115. A 55-year-old woman with interstitial lung disease presented with exertional dyspnea and bilateral lower limb edema. Physical examination revealed an elevated JVP with a prominent v- wave, pulsating hepatomegaly, a left parasternal heave, and a loud pulmonary component of the second heart sound (P2). Which of the following is the most expected additional finding on heart auscultation? A. Mitral regurgitation B. Aortic regurgitation C. Tricuspid regurgitation D. Pulmonary regurgitation Answer: C 0116. A 65-year-old man came to the Emergency Room after his wife said he went to bed without complaints but woke up 2 hours later unable to walk or speak. He is known to have type 2 diabetes mellitus, hypertension, dyslipidemia, and osteoarthritis, and is on insulin, metformin, candesartan, hydrochlorothiazide, aspirin, and atorvastatin. Examination reveals he is conscious but disoriented. Right side of his body has power of 2/5 (see image). Which of the following is the most appropriate next step? A. Clopidogrel (Plavix) B. Rivaroxaban C. Aspirin D. rtPA (recombinant tissue plasminogen activator) Answer: D 0117. A 73-year-old woman is brought to the emergency room by family members who noticed a change in her level of consciousness for 1 day. She is known to have type 2 diabetes mellitus, hypertension, ischemic heart disease, and liver cirrhosis. Examination reveals she is drowsy, with raised JVP, positive shifting dullness, and +2 lower limb edema (see lab results). Which of the following is the most appropriate next step in management? A. Furosemide B. Normal saline C. 0.45% normal saline D. D5% normal saline Answer: A 0118. A 63-year-old man presents to the clinic with fatigue and tiredness. No fever, shortness of breath, chest pain, abdominal pain, or vomiting. He is known to have type 2 diabetes mellitus, hypertension, dyslipidemia, and chronic kidney disease stage IV. Examination reveals a well- looking patient, not in distress, with normal JVP and no lower limb edema (see lab results). Which of the following is the most appropriate next step in management? A. Observe B. Start Iron C. Start erythropoietin D. Packed RBC transfusion Answer: C 0119. A 70-year-old gentleman known to have liver cirrhosis with esophageal varices underwent endoscopic variceal ligation 1 month ago. Which of the following is the best long-term intervention to decrease the risk of recurrent variceal bleeding? A. B blocker B. H2 blocker C. Proton pump inhibitor D. No further intervention Answer: A 0120. A 33-year-old woman, who is known to have bronchial asthma and chronic sinusitis, is complaining of worsening shortness of breath and a cough that disturbs her sleep. She denied any fever, headache, or chest pain. Her medications include salbutamol MDI prn and paracetamol. Chest examination reveals bilateral expiratory wheezes. Which of the following should be added to the management plan? A. Montelukast B. Budesonide MDI C. Oral prednisone D. Ipratropium bromide MDI Answer: B 0121. A 65-year-old man, a heavy smoker, known to have hypertension and COPD on Ventolin, Anoro Ellipta (Umeclidinium bromide/vilanterol), and lisinopril, presents with worsening shortness of breath for 4 months. There has been no change in his sputum amount or purulence, and he has had no fever, PND, or orthopnea. He was not admitted to the hospital for more than a year. Examination reveals decreased breath sounds without added sounds (see lab results and report). Chest CT 6 months ago: Diffuse emphysematous changes. Which of the following is the best intervention to improve his shortness of breath? A. Home oxygen B. Add inhaled long-acting B agonist C. Volume reduction surgery D. Add inhaled corticosteroid Answer: A 0122. A 55-year-old man known to have psychiatric disease and hypertension was admitted to the hospital due to poor nutrition at home, as he was refusing food. During hospitalization, he became agitated, so they increased the dose of Haloperidol. Suddenly, he became confused and began sweating. He was shifted to the ICU, started on IV fluids, and intubated (see lab result). Which of the following should be added to the management plan? A. Naloxone B. Glucagon C. Meropenem D. Bromocriptine Answer: D 0123. A 76-year-old man, known to have type 2 diabetes mellitus, hypertension, and dyslipidemia, presented to the clinic with exertional shortness of breath for 1 year. He has to slow down his walk and stop after 10 meters. He denied any history of syncope, chest pain, or palpitation. Precordium examination revealed a forceful apex beat and a crescendo- decrescendo ejection systolic murmur radiating to the carotid. Which of the following is the most likely diagnosis? A. Mitral stenosis B. Aortic stenosis C. Pulmonary stenosis D. Aortic regurgitation Answer: B 0124. A 62-year-old woman known to have type 2 diabetes mellitus, hypertension, dyslipidemia, and ischemic stroke 1 month ago, presented to the Emergency Room complaining of central stabbing continuous chest pain for 3 hours. She denied any dyspnea, palpitation, or change in consciousness. The patient is taking insulin, lisinopril, metoprolol, lipitor, and aspirin. Examination revealed she is in pain. She was started on oxygen, morphine, and sublingual nitroglycerin (see image). Which of the following is the most appropriate next step? A. Give IV magnesium B. Do CT angiography C. Urgent coronary angiography D. Give thrombolytic medication Answer: C 0125. A 58-year-old man known to have type 2 diabetes mellitus, hypertension, and ischemic heart disease. His medications are metformin, sitagliptin, aspirin, lipitor, and lisinopril. He works as a car mechanic. For the last 4 months, he has started to notice exertional shortness of breath that interferes with his job but does not limit him from doing it, provided that at rest, he is fine without any shortness of breath. Which of the following is the New York Heart Association (NYHA) class of his shortness of breath? A. Class I B. Class II C. Class III D. Class IV Answer: B 0126. A 35-year-old woman who had an uncomplicated vaginal delivery 2 weeks ago presented to the Emergency Room complaining of chest pain for 1 day, associated with exertional shortness of breath, no cough, no fever, no orthopnea, and no PND. Examination revealed an ill-looking woman (see reports). Pulmonary function test: FEV1 90% (80%-120%) FVC 95% (80%-120%) TLC 95% (80%-120%) DLCO 40% (60%-120%) Which of the following is the most likely cause of her shortness of breath? A. Pneumonia B. Heart failure C. Acute bronchitis D. Pulmonary embolism Answer: D 0127. A 38-year-old woman who is not known to have any medical illness is referred to the clinic because of a chest x-ray that showed prominent hilar structures bilaterally. She is having on- and-off cough but denies any history of shortness of breath, sputum production, chest pain, skin lesions, weight loss, fever, or contact with sick people. She is not a smoker. Examination reveals clear chest bilaterally (see lab results and reports). CT chest: Confirmed bilateral hilar lymphadenopathy, otherwise lung parenchyma was clear. CT-guided biopsy: Showed noncaseating granuloma. Which of the following is the most appropriate next step in management? A. Observe B. Start prednisone C. Start azithromycin D. Start anti-tuberculosis therapy Answer: A 0128. A 68-year-old woman known to have type 2 diabetes mellitus, hypertension, and end- stage renal disease on hemodialysis complains of shortness of breath and increasing bilateral lower limb swelling. She denies any fever, chest pain, cough, abdominal pain, or vomiting. Examination reveals an ill-looking patient with high JVP and +3 lower limb edema. ECG was ordered (see lab result). Which of the following is the most likely ECG finding? A. Peaked T wave B. Peaked P wave C. Short PR interval D. Narrow QRS complex Answer: A 0129. A 78-year-old man known to have type 2 diabetes mellitus, hypertension, and chronic kidney disease is complaining of fatigue and nausea. He denies any change in consciousness, seizures, abdominal pain, vomiting, chest pain, or shortness of breath. He is on insulin, lisinopril, amlodipine, erythropoietin, and vitamin D. Examination reveals that he is drowsy but not in distress (see lab results). Which of the following is the most appropriate next step in management? A. NaHCO3 B. D5 water C. Diuretics D. Calcium gluconate Answer: D 0130. A 33-year-old woman known to have systemic sclerosis with normal kidney function, diagnosed 2 years ago, is on pantoprazole. She complains of drowsiness and visual changes, denies fever, cough, shortness of breath, chest pain, seizures, abdominal pain, or vomiting. Examination reveals an unwell patient with raised JVP, clear chest, and +3 lower limb edema (see lab result). Which of the following is the most appropriate management? A. Lisinopril B. Bisoprolol C. Amlodipine D. Hydrochlorothiazide Answer: A 0131. A 45-year-old woman known to have end-stage renal disease secondary to type 1 diabetes mellitus came to the clinic for a routine follow-up. She had no complaints but was found to have abnormal blood results (see lab results). Which of the following is the most appropriate management? A. Sevelamer B. Calcitriol C. Cinacalcet D. 25-hydroxy vitamin D Answer: B 0132. A 38-year-old woman known to have type 2 diabetes mellitus just returned from Makkah. She complains of fever, lethargy, and headache. She has no cough, chest pain, shortness of breath, abdominal pain, nausea, vomiting, or skin rash. Examination reveals a drowsy patient with positive neck stiffness (see report). Lumbar puncture: Low glucose, markedly increased protein, WBC 9000 with predominantly neutrophils. Which of the following is the most appropriate medication to start? A. Cefotaxime and vancomycin B. Ampicillin and cefotaxime C. Ceftriaxone and vancomycin D. Ceftriaxone and azithromycin Answer: C 0133. A 28-year-old woman, not known to have any medical illness, complains of increased sleepiness, fatigue, and constipation for the last few months. She is on a very healthy diet, is not a smoker, and does not drink alcohol. Examination reveals an overweight patient, not in distress, with no lymphadenopathy or masses (see lab results). Which of the following is the most likely diagnosis? A. Graves' disease B. Subacute thyroiditis C. Sick euthyroid syndrome D. Hashimoto's thyroiditis Answer: D 0134. A 65-year-old man was brought to the Emergency Room by his wife after she noticed he was confused and behaving in a strange way. He had no fever, cough, shortness of breath, chest pain, headache, abdominal pain, vomiting, or loss of consciousness. He is known to have type 2 diabetes mellitus, hypertension, dyslipidemia, and osteoarthritis, and is on insulin, metformin, candesartan, hydrochlorothiazide, aspirin, and atorvastatin. Examination reveals that he is conscious but disoriented. Left leg power is 2/5, while the other limbs are 5/5. There is decreased sensation in the left leg. Other neurological exams are normal. Which of the following is the most likely explanation for his symptoms and signs? A. Migraine B. Right MCA stroke C. Right ACA stroke D. Basilar artery stroke Answer: C 0135. A 22-year-old man presented to the Emergency Room with left leg swelling and pain that started 2 days ago and is getting worse. It involves the whole left leg. He also complains of chest tightness and shortness of breath. Examination reveals left lower limb pitting edema extending to the thigh (see report). ECG: Sinus tachycardia. Which of the following is the most appropriate management at this time? A. Aspirin B. Clopidogrel C. Warfarin D. Unfractionated heparin Answer: D 0136. A 45-year-old man presented to the Emergency Room with palpitations. Initially, he was conscious and oriented. His vital signs were as shown. After some time, he became less responsive, and his blood pressure dropped to 80/45 mmHg (see report). ECG: Supraventricular tachycardia (SVT). Which of the following is the most appropriate immediate action? A. Pacemaker B. Cardioversion C. Propranolol D. Radiofrequency ablation Answer: B 0137. A 16-year-old boy came to the Emergency Department complaining of exertional dyspnea and easy fatiguability for the past few weeks. His parents admitted that he used to be a sick child requiring multiple hospital admissions and frequent blood transfusions. His parents are related. He was found to be pale, jaundiced, and have hepatosplenomegaly, with a scar from laparoscopic cholecystectomy (see lab results). Which of the following has the highest diagnostic value? A. Peripheral blood film B. Sickle cell saturation C. Bone marrow biopsy D. Hemoglobin electrophoresis Answer: D 0138. A 70-year-old man presented to the Emergency Department complaining of increasing shortness of breath and a productive cough for the past 5 months. He has been a smoker for over 50 years. Over the course of 5 months, he complained of malaise, night sweats, fever, and has lost 10 kg over the last 3 months. Which of the following is the most likely diagnosis? A. Lung carcinoma B. Pneumonia C. Bronchial asthma D. Pulmonary embolism Answer: A 0139. A 60-year-old man was admitted to the hospital because of community-acquired pneumonia. On hospital day 3, he developed increasing shortness of breath and became severely hypoxic, necessitating mechanical ventilation with a PaO2/FiO2 of 180 (see report). Chest X-ray: Bilateral lung infiltrates. Which of the following is the most likely diagnosis? A. Myocardial infarction (MI) B. Acute respiratory distress syndrome (ARDS) C. Pleural effusion D. Pulmonary hypertension Answer: B 0140. A 25-year-old college student came for a routine checkup. He does not have any active complaints. Examination reveals a diastolic murmur heard at the base of the heart, which increased when the patient leaned forward in full expiration. Which of the following is the most likely diagnosis? A. Aortic stenosis B. Mitral regurgitation C. Aortic regurgitation D. Tricuspid regurgitation Answer: C 0141. A 39-year-old woman presented to the family medicine clinic for counseling as she is planning to get pregnant. She gave a history of hypertension and is controlled on an angiotensin-converting enzyme inhibitor (ACEI). Her physician asked her to change the medication as it is contraindicated during pregnancy. Which of the following is the most likely effect of ACEI on pregnancy? A. Increases the rate of preeclampsia B. Increases the rate of fetal malformation C. Increases the rate of premature rupture of membranes D. Leads to a significant drop in blood pressure during pregnancy Answer: B 0142. A 23-year-old man known to have Hodgkin's lymphoma, undergoing chemotherapy, developed tumor lysis syndrome. Which of the following is the most likely expected disturbance? A. Hypokalemia and hypocalcemia B. Hypokalemia and hypercalcemia C. Hyperkalemia and hypocalcemia D. Hyperkalemia and hypercalcemia Answer: C 0143. A 50-year-old man known to be an alcoholic presented with progressive abdominal distention and yellow discoloration of the sclera. The patient gave a history of vague abdominal pain mainly at the epigastrium. Fluid thrill was found upon examination, suggestive of ascites. Diagnostic paracentesis was done (see lab results). Which of the following is the most likely cause of his ascites? A. Nephrotic syndrome B. Acute pancreatitis C. Portal hypertension D. Congestive heart failure Answer: C 0144. A 45-year-old man was recently diagnosed with hypertension. He has a family history of ischemic heart disease. An angiotensin receptor II blocker was prescribed to control his blood pressure and reduce the risk of ischemic heart disease. Which of the following is the most likely prescribed agent? A. Carvedilol B. Irbesartan C. Perindopril D. Amiodarone Answer: B 0145. A 42-year-old man presented to the hospital complaining of shortness of breath, fever, and fatigue for the past few days. Examination showed he has high JVP, pulsus paradoxus, and distant heart sounds. Lung auscultation was clear (see reports). ECG: Low voltage. Chest X-ray: An increase in cardiac silhouette. Which area of the heart is most likely involved? A. Valves B. Pericardium C. Myocardium D. Coronary arteries Answer: B 0146. A 22-year-old woman was brought to the Emergency Room after ingesting 50 tablets of extra strength acetaminophen (500 mg each tablet) 3 hours ago. The patient is conscious and oriented (see lab results). Which of the following is the most appropriate next step? A. Hemodialysis B. N-acetylcysteine and charcoal C. Close monitoring of liver function tests D. Intubation and mechanical ventilation Answer: B 0147. A 43-year-old man presented to the hospital with right leg swelling and pain. He has a history of pulmonary embolism 5 years ago and is currently not taking any medications. Doppler ultrasound confirmed the diagnosis of proximal Deep Vein Thrombosis (DVT). Which of the following is the most appropriate management? A. Aspirin B. Thrombolysis C. Long-term anticoagulation D. Anticoagulation for 2 months Answer: C 0148. A 36-year-old man, previously healthy, presented to primary healthcare complaining of cough, fatigue, sore throat, and body aches. The cough is productive with whitish sputum; on a few occasions, there were some blood streaks. Chest examination was within normal limits (see report). Chest X-ray: Normal, without evidence of consolidation. Which of the following is the most likely diagnosis? A. Bronchitis B. Pulmonary edema C. Goodpasture syndrome D. Hereditary hemorrhagic telangiectasia Answer: A 0150. A 42-year-old homeless man was recently released from prison. He presented to the Emergency Room with a fever, productive cough, weight loss, and night sweats over the past 3 months (see report). Chest X-ray: Left lung apical infiltration. Sputum sample was collected and sent to the lab. Which of the following tests has the highest diagnostic value? A. Thin blood film B. Congo red stain C. Thick blood film D. Ziehl-Neelsen stain Answer: D 0151. A 35-year-old man comes to the Emergency Room after falling while running a marathon. He has dark urine that is dip-stick positive for large blood, but no RBCs on microscopic analysis. Which of the following is the most likely diagnosis? A. Hemorrhagic cystitis B. Glomerulonephritis C. Rhabdomyolysis D. Pyelonephritis Answer: C 0152. A 35-year-old man comes to the Emergency Room after falling while running a marathon. He has dark urine that is dip-stick positive for large blood, but no RBCs on microscopic analysis. Which of the following is the most appropriate management? A. Diuresis B. Hydration C. Antibiotics D. Antihypertensive Answer: B 0153. A 50-year-old woman presents with nausea, vomiting, and muscle aches. She is 6 days post-operative and is unable to pass stool, and has abdominal distension (see lab results). Which of the following is the most appropriate next step in management? A. ECG B. Urine potassium C. Stool potassium D. Urine osmolality Answer: A 0154. A patient with acute leukemia recently underwent allogenic bone marrow transplantation. After 2 weeks, she was neutropenic and developed a fever and cough. She was diagnosed with invasive aspergillosis based on radiological and microbiological assessment. Which of the following is the most appropriate treatment? A. Rifampicin B. Valaciclovir C. Tigecycline D. Voriconazole Answer: D 0155. A 32-year-old woman presents with pain in her hands and knees on and off for 2 months. She had 3 abortions in the first trimester. Examination reveals a rash over the nose bridge, swelling and tenderness of the small joints of the hands, and a warm, tender, erythematous swelling of the right calf that is 2 cm larger in circumference than the left (see lab results). Which of the following is the most appropriate venous thromboembolism prophylaxis in her case? A. Lifelong warfarin therapy B. Long-term enoxaparin therapy C. Long-term unfractionated heparin therapy D. Lifelong non-vitamin K anticoagulant Answer: A 0156. A 46-year-old woman presented to the Emergency Room with back pain for 3 weeks associated with fever. She was sent home with NSAIDs and bed rest recommendations but returned after 2 days with the inability to move her legs. Examination revealed a febrile patient with tenderness in the mid-back and upper motor neuron signs in the lower limbs associated with weakness 3/5 (see lab results and report). MRI spine: Diffuse discitis in T6 vertebrae. Which of the following is the most appropriate next step in management? A. Tuberculin (PPD) test B. Brucella titers C. Biopsy of T6 vertebrae D. Perform a bone marrow aspiration for culture Answer: B 0157. An 85-year-old man admitted to the Intensive Care Unit following a massive stroke suddenly developed new weakness. Physical examination confirmed papilledema (see report). Emergency CT brain: Hemorrhagic transformation. Which of the following is the most definitive treatment? A. Intravenous dexamethasone B. Administration of mannitol C. Craniotomy and decompression D. Administering high-frequency ventilation Answer: C 0158. A 59-year-old woman admitted with an infected bedsore and had debridement 2 days ago. She suddenly developed mental confusion and poor response to commands. Which of the following is the most likely cause of her deterioration? A. Septic shock B. Anaphylactic shock C. Cardiac shock D. Acute pulmonary embolism Answer: A 0159. A 22-year-old prisoner presented with fever, productive cough, and weight loss for 3 weeks. He was admitted to isolation as a case of open tuberculosis (TB), and the next day, he was discovered to be HIV positive. Which of the following is the most appropriate management? A. Start both HIV and TB therapy B. Consider pneumocystis pneumonia C. Start HIV therapy first until CD4 count improves D. Start anti-TB therapy alone to prevent immune reconstitution inflammatory syndrome Answer: D 0160. A 70-year-old man complains that he cannot remember things, sometimes forgetting names of friends, famous community figures, and telephone numbers. His wife fears he has Alzheimer’s disease. Physical examination is unremarkable apart from tender knee joints (see lab results). Which of the following is the most likely diagnosis? A. Alzheimer’s disease B. Lewy body dementia C. Benign forgetfulness D. Hypertensive encephalopathy Answer: C 0161. A 39-year-old man diagnosed with pancreatic cancer develops a swollen left lower limb during hospitalization. Examination reveals a tender, erythematous swelling affecting the left leg. Which of the following is the most likely diagnosis? A. Lymphedema B. Cellulitis C. Ruptured Baker's cyst D. Deep vein thrombosis Answer: D 0162. A 16-year-old boy presents with a 4-day history of a rash over his legs associated with abdominal pain and aches in both knees. He had one bout of bloody loose bowel motions. Physical examination reveals an erythematous palpable rash extending up to his buttocks (see reports). Urinalysis: Moderate proteinuria Ultrasound kidneys: Normal Which of the following is the most likely diagnosis? A. Churg-Strauss syndrome B. Hemolytic uremic syndrome C. Henoch-Schönlein purpura D. Thrombotic thrombocytopenic purpura Answer: C 0163. A 20-year-old woman, recently married, presents with burning micturition for one week. Physical examination reveals suprapubic tenderness (see lab results). Which of the following is the most appropriate antibiotic? A. Oral nitrofurantoin B. Oral fluoroquinolone C. Gentamicin intravenously D. Oral beta-lactam antibiotic Answer: A 0164. A 65-year-old woman with long-standing chronic renal failure presents with aches and pains for 1 month. Physical examination is unremarkable apart from general pallor (see lab results). Which of the following is the most appropriate next step in management? A. Oral calcitriol B. Oral vitamin D3 C. Oral vitamin D2 D. Calcitonin nasal spray Answer: A 0165. A 38-year-old woman, mother of three, presents with fatigue and weight loss for the last few months. She was unable to breastfeed her last child and her menses did not return after giving birth. She has cold intolerance and loss of appetite. Examination reveals a croaky voice with delayed relaxation phase of her ankle jerks (see lab results). What is the next step in management? A. Give conjugated estrogen B. Request Thyroid Stimulating Hormone (TSH) level C. Correct hyponatremia with hypertonic saline D. Treat with levothyroxine 50 mcg per day Answer: B 0166. A 22-year-old woman presents with recurrent seizures and cognitive dysfunction. She is subsequently diagnosed with glioblastoma multiforme (see lab results). Which of the following is the most likely cause of her hyponatremia? A. Salt-wasting nephropathy B. Excessive water consumption C. Increased consumption of soft drinks D. Syndrome of inappropriate ADH secretion Answer: D 0167. A 60-year-old man known to have systemic hypertension, gout, and dyslipidemia presented to the Emergency Room 5 days ago with signs of heart failure and was diagnosed with congestive heart failure and new-onset diabetes mellitus. He was stabilized and discharged on Lasix, aspirin, Plavix, and metformin. Yesterday, he came to the ER with severe pain and redness in the first metatarsophalangeal joint. Examination confirms tenderness and swelling of that joint. Which of the following medications has most likely caused this new complaint? A. Lasix B. Plavix C. Aspirin D. Metformin Answer: A 0168. A 26-year-old woman complains of marked fear whenever dealing with unfamiliar people, as she is afraid of embarrassment. She was referred to a local psychologist for assessment and management. Which of the following will benefit from cognitive behavioral therapy? A. Schizophrenia B. Social phobia C. Postpartum psychosis D. Depression with hallucination Answer: B 0169. A 69-year-old man is diagnosed recently with lymphoma and started on chemotherapy. On day 7 post 4th cycle, he presented to the Emergency Room with fever. Examination precludes any underlying infection (see lab results). Which of the following is the most appropriate treatment? A. NSAID and antifungal therapy B. Oral antibiotics and send home C. Blood culture and wait for sensitivity D. Empiric IV antibiotics in a hospital setting Answer: D 0170. A 55-year-old heavy smoker, smoking 4 packs a day, complains of occasional chest pain on exertion, which is relieved by rest. He presents to the clinic seeking help to stop smoking. Physical examination reveals an anxious patient with nicotine stains on the fingers and peripheral cyanosis. His jugular venous pulse is raised 8 cm. The rest of the exam is unremarkable. Which of the following is the most appropriate initial therapy? A. Bupropion B. Motivational support C. Varenicline (Chantix) D. Nicotine replacement therapy Answer: D 0171. A 20-year-old woman is referred to Neurology Clinic because of an unsteady gait and transient loss of vision during a sports game at university. She has no history of fever, sweating, or recent trauma, and no past medical or drug history (see report). MRI Brain: Multiple demyelinating hyperintense lesions are seen distributed in the periventricular white matter and callososeptal regions bilaterally. Which of the following is the most likely diagnosis? A. Brain vasculitis B. CNS lymphoma C. Multiple sclerosis D. Vitamin B12 deficiency Answer: C 0172. A 48-year-old woman with rheumatoid arthritis presents to the clinic with a 2-week history of joint pain and increased morning stiffness lasting almost 45 minutes. Her current medications include prednisolone 5 mg daily and methotrexate 15 mg weekly. Her examination reveals multiple swollen and tender joints (see lab results). Which of the following is the most appropriate treatment to be added? A. Ibuprofen B. Azathioprine C. Cyclosporine D. Biologic agent Answer: D 0173. A 56-year-old man with chronic renal failure for 5 years presents to the clinic with fatigue for 3 months. His physical examination is unremarkable (see lab results). Which of the following is the most appropriate treatment? A. Blood transfusion B. Intravenous iron therapy C. Subcutaneous erythropoietin D. Oral folic acid supplementation Answer: C 0174. A 58-year-old woman with liver cirrhosis presents to the emergency with hematemesis (see report). Upper GI endoscopy: Bleeding esophageal varices Which of the following is the most appropriate intravenous pharmacological treatment? A. Octreotide B. Propranolol C. H2 receptor blockers D. Proton pump inhibitors (PPIs) Answer: A 0175. A 60-year-old man with liver cirrhosis presents to the Emergency Room with increasing ascites. He has no history of fever or decreased level of consciousness. His medications include spironolactone 50 mg/day and furosemide 40 mg/day. His physical examination confirms the presence of large ascites (see lab results). Which of the following is the most appropriate next step in management? A. Intravenous diuretics B. Therapeutic paracentesis C. Increase the dose of furosemide D. Transjugular intrahepatic portosystemic shunt (TIPS) Answer: B 0176. A 21-year-old man with a recurrent history of wheezing and exertional shortness of breath when participating in exercises presents to the Emergency Room with increasing shortness of breath after an aerobic class. Examination reveals that he is having obvious difficulty in breathing with diffuse bilateral wheezes. Which of the following is the most effective treatment to be started in the emergency? A. Intravenous corticosteroid B. Inhaled salbutamol C. Inhaled salmeterol D. Inhaled fluticasone Answer: B 0177. A 28-year-old man presents to the Emergency Room with shortness of breath on exertion that started after exercise. He has a history of intermittent wheezing in response to exercise. His chest examination reveals diffuse wheezes. Which of the following is the most appropriate treatment? A. Inhaled salbutamol B. Inhaled fluticasone C. Intravenous magnesium D. Intravenous hydrocortisone Answer: A 0178. A 26-year-old man with type I diabetes mellitus on insulin presents to the clinic for regular follow-up. He is asymptomatic (see lab results). Which of the following is the most appropriate treatment? A. Diuretics B. Calcium channel blockers C. Beta-adrenergic blocking agents D. Angiotensin-converting enzyme inhibitor Answer: D 0179. A 38-year-old woman known to have systemic lupus erythematosus (SLE) presents for regular follow-up. She is feeling unwell. Examination reveals multiple joint swelling and oral ulcer. Her 24-hour urine collection showed proteinuria of 1.5 g/24h (see lab results). Which of the following tests will confirm her kidney disease? A. PET scan B. Kidney biopsy C. Renal angiogram D. 24-hour Na and K collection Answer: B 0180. A 35-year-old woman was referred to the clinic because she was found to have proteinuria. She is complaining of fatigue and lower limb swelling. Her examination reveals lower limb pitting edema up to the knees bilaterally. Her systemic examination is unremarkable (see lab results). Which of the following is the most diagnostic test to establish the diagnosis? A. Kidney biopsy B. Kidney ultrasound C. Complements level D. Antinuclear antibody Answer: A 0181. A 36-year-old woman who is hypertensive on hydrochlorothiazide and losartan presents to the clinic because of the absence of her menstrual cycle for 2 months. She is asymptomatic. Her physical examination is unremarkable. A pregnancy test was done and was positive (see lab results). Which of the following is the most appropriate treatment adjustment? A. Continue same antihypertensive agents B. Discontinue all antihypertensive agents C. Discontinue losartan and start methyldopa D. Discontinue losartan and start enalapril Answer: C 0182. A 60-year-old man presents to the Emergency Room with symptomatic hyponatremia. Which of the following is the most serious consequence of rapid correction of hyponatremia? A. Hypokalemia B. Hypernatremia C. Volume overload D. Central pontine myelinolysis Answer: D 0183. A 29-year-old man presents to the Emergency Room with a 2-day history of fever and painful skin eruption over the left side of his chest. His physical examination reveals a skin eruption consisting of maculopapular lesions and vesicles with some skin crusting over the left side of his chest. His systemic examinations are unremarkable. Which of the following is the most appropriate action to confirm the diagnosis? A. Skin biopsy B. Blood culture C. Swab from skin lesion for culture D. No further investigation is required Answer: D 0184. A 20-year-old woman is brought to the Emergency Room with a 2-day history of headache, fever, photosensitivity, lethargy, and skin eruption. Her examination reveals a drowsy woman with neck rigidity but no focal neurological findings. She has a purpuric skin rash over her extremities (see lab results). Which of the following is the most appropriate investigation to establish the diagnosis? A. Skin biopsy B. Blood culture C. Lumbar puncture D. Meningococcal serology Answer: C 0185. A 35-year-old woman is evaluated in the clinic for thyroid enlargement. She is asymptomatic and has no significant past medical history. Her examination reveals a firm, mobile, non-tender nodule at the lower pole of the right lobe of the thyroid gland. The left lobe is normal to palpation. Which of the following investigations is the most appropriate initial test? A. Thyroid Tc99 scan B. Fine needle aspiration (FNA) of the nodule C. Triiodothyronine (T3) level D. Thyroid stimulating hormone (TSH) Answer: D 0186. A 55-year-old man with acute myeloid leukemia on chemotherapy presents to the Emergency Room with a 1-day history of fever. His physical examination is unremarkable. His chest radiograph and urinalysis are normal. Blood and urinary cultures are taken (see lab results). Which of the following is the most appropriate next step in management? A. IV antibiotics B. Close observation C. IV antibiotics and antiviral agents D. Oral antibiotics and antifungal agents Answer: A 0187. A 50-year-old man presents to the Emergency Room with fever, progressive shortness of breath, and productive cough for 3 days. Examination is unremarkable except for coarse crackles at the right lower lung (see lab results and reports). Blood culture: Gram-positive cocci. Intravenous antibiotics are started. Which of the following is the most appropriate next step in treatment? A. Initiate hemodialysis B. Initiate dobutamine infusion C. Administer 1000 mL normal saline fluid bolus D. Transfuse one unit of packed red blood cells Answer: C 0188. A 54-year-old man presents to the Emergency Room with an acute exacerbation of COPD. Examination reveals the use of accessory muscles of breathing, diminished gag reflex, decreased breath sounds with wheezing heard throughout both lung fields. Oxygen, glucocorticoids, antibiotics, and nebulizers are started (see lab results). Which of the following is the most appropriate next step in management? A. Start noninvasive ventilation B. Close observations for the next 4-6 hours C. Intubate and start mechanical ventilation D. Increase oxygen to 100% by non-rebreather mask Answer: C 0189. A 35-year-old man, who was medically free, was admitted to the Intensive Care Unit with massive pulmonary embolism. Which of the following is the most appropriate initial treatment? A. Warfarin B. Thrombolysis C. Low molecular heparin D. Direct thrombin inhibitor (dabigatran) Answer: B 0190. A 32-year-old woman presents to the clinic with a 3-month history of lower back pain associated with morning stiffness lasting almost 1 hour, as well as joint pain affecting both knees and ankles bilaterally. She denied any history of skin rash or mucosal ulcerations. No history of fever. Clinical examination reveals limited lumbar spine movements and active arthritis at both knees bilaterally. She also has tenderness over the Achilles tendon (see lab results). Which of the following is the most likely diagnosis? A. Reactive arthritis B. Rheumatoid arthritis C. Ankylosing spondylitis D. Crystal deposition arthritis Answer: C 0191. A 38-year-old woman presents to the Emergency Department with increased shortness of breath on exertion, palpitations, and fatigue for one week. Physical examination reveals pale conjunctiva and splenomegaly. Peripheral blood smear shows spherocytosis (see lab results). Which of the following is the most appropriate treatment? A. Blood transfusion B. Hydroxyurea therapy C. Corticosteroid therapy D. Parenteral iron therapy Answer: C 0192. A 30-year-old woman presents to the clinic with a 3-month history of palpitations and fatigue. Physical examination is unremarkable (see lab results). Which of the following is the most appropriate treatment? A. Blood transfusion B. Parenteral iron therapy C. Oral folic acid therapy D. Oral iron supplementation Answer: D 0193. A 28-year-old man presents to the Emergency Department with a 2-week history of bloody diarrhea, lower abdominal cramps, and tenesmus. Physical examination reveals abdominal distension with mild tenderness (see lab results). Which of the following is the most appropriate investigation to establish the diagnosis? A. Colonoscopy B. Abdominal CT scan C. Clostridium difficile toxins D. Stool for culture and sensitivity Answer: A 0194. A 50-year-old man with ischemic heart disease and diabetes presented to the emergency with severe pneumonia and was admitted to the intensive care unit (ICU). He was treated with a course of antibiotics. Three days after admission to ICU, he develops hypotension treated with hydration and inotropes. His liver function at the day of admission was normal, but the repeated one is abnormal (see lab results and report). Abdominal ultrasound: Unremarkable. Which of the following is the most likely diagnosis? A. Ischemic hepatitis B. ICU-related jaundice C. Intravascular hemolysis D. Acalculous cholecystitis Answer: A 0195. A 42-year-old man presents to the Emergency Room with productive cough, fever, and dyspnea for 3 days. Chest examination reveals evidence of consolidation at the right lower lung field. Baseline laboratory studies, two blood cultures, and sputum samples for cultures and gram stain are obtained. Which of the following is the most appropriate empiric antimicrobial regimen? A. Meropenem and azithromycin B. Ceftriaxone and azithromycin C. Vancomycin and azithromycin D. Oseltamivir and azithromycin Answer: B 0196. A 60-year-old man with hypertension presents to the Emergency Room with sudden onset of retrosternal chest pain, dyspnea, and palpitations for 2 hours. His past medical history is significant for a non-hemorrhagic stroke 2 months ago (see lab results and report). ECG: ST-segment elevation in leads II, III, and aVF. Which of the following is the most appropriate treatment? A. Fibrinolytic therapy B. Aspirin therapy C. Unfractionated heparin D. Percutaneous coronary intervention (PCI) Answer: D 0197. A 55-year-old man presents to the clinic with progressive shortness of breath on exertion for 8 months. He has no history of chest pain or other systemic manifestations. Physical examination reveals an early diastolic murmur at the left upper sternal border that is augmented when sitting forward (see reports). Chest radiography: Cardiomegaly. ECG: Left ventricular hypertrophy. Which of the following is the most likel